Question:

As government agencies, faced with budget difficulties, reduce their funding for scientific research, a greater amount of such research is being funded by private foundations. This shift means that research projects likely to produce controversial results will almost certainly comprise a smaller proportion of all funded research projects, since private foundations, concerned about their public image, tend to avoid controversy.
Which of the following is an assumption on which the argument depends?

Show Hint

To test if a statement is a necessary assumption, use the "Negation Test." Negate the statement and see if the argument falls apart. If we negate (E), we get: "Government agencies are NOT more willing (i.e., are equally or less willing) than private foundations to fund controversial research." If this is true, then shifting funding from the government to private foundations would not cause a decrease in controversial projects, and the argument's conclusion would be invalid.
Updated On: Oct 4, 2025
  • Only research that is conducted without concern for the possibility of generating controversy is likely to produce scientifically valid results.
  • Private foundations that fund scientific research projects usually recognize that controversial results from those projects cannot always be avoided.
  • Scientists who conduct research projects funded by private foundations are unlikely to allow the concerns of the funding organizations to influence the manner in which they conduct the research.
  • Many government agencies are more concerned about their public image than are most private foundations.
  • Government agencies are more willing than are private foundations to fund research projects that are likely to produce controversial results.
Hide Solution
collegedunia
Verified By Collegedunia

The Correct Option is

Solution and Explanation

Step 1: Understanding the Concept:
This is an assumption question. The argument concludes that a shift in funding from government to private foundations will lead to less controversial research being funded. We need to find the unstated premise that connects the evidence (shift in funding, private foundations avoid controversy) to the conclusion (less controversial research overall).
Step 2: Detailed Explanation:
The structure of the argument is:

Premise 1: Funding is shifting from government agencies to private foundations.
Premise 2: Private foundations avoid funding controversial research.
Conclusion: Therefore, the proportion of controversial research will decrease.
The argument implicitly assumes that the group losing funding (government agencies) was \textit{more} likely to fund controversial research than the group gaining funding (private foundations). If the government was just as risk-averse as private foundations, then shifting the funding source wouldn't change the proportion of controversial research. The argument only works if there's a difference in their willingness to fund such projects. Let's analyze the options:

(A) This goes too far. The argument is about the \textit{amount} of controversial research, not its scientific validity.
(B) This weakens the argument by suggesting private foundations are not completely averse to controversy.
(C) This also weakens the argument. If scientists ignore the foundations' concerns, the shift in funding might not affect the type of research conducted.
(D) This is the opposite of what needs to be assumed. The argument assumes private foundations are \textit{more} concerned with their public image (and thus avoiding controversy) than the government.
(E) This directly states the missing link. The argument's conclusion that controversial research will decrease only follows if the original funders (government) were more willing to fund it than the new funders (private foundations). This comparison is essential for the conclusion to be valid.
Step 3: Final Answer:
The argument depends on the assumption that government agencies are more willing to fund potentially controversial research than private foundations are. Option (E) correctly identifies this necessary assumption.
Was this answer helpful?
0
0

Top Questions on Critical Reasoning

View More Questions

Questions Asked in GRE exam

View More Questions